Improper integral using residues

Click For Summary
The discussion centers on evaluating the improper integral ∫(√x)/(x²+1)dx from 0 to ∞ using residue theory. A substitution of √x = t leads to the integral ∫(2t²)/(t⁴+1)dt, which simplifies the problem. The correct evaluation yields a result of π/√2, indicating that a branch cut along the x-axis is unnecessary. A mistake in the original substitution was identified, where a factor of 2 was omitted. The conversation emphasizes the importance of providing hints rather than complete solutions in homework discussions.
player1_1_1
Messages
112
Reaction score
0

Homework Statement


\int\limits^{ +\infty }_{0}\frac{ \sqrt{x} \mbox{d} x }{ x^2+1 }

The Attempt at a Solution


I make substitution \sqrt{x}=t and then
\int^{ +\infty }_{0}\frac{ t^2 \mbox{d} t }{ t^4+1 }
and now this function is odd, so I make a half circle and count residues yeah?
 
Physics news on Phys.org
Since you have \sqrt{x} I believe you have a branch cut along the x-axes. Your integral should be:

\oint\frac{z^{1/2}}{z^2+1}dz

I'd try to solve it and I'll get back to you :D

EDIT: Yep I'm right, solved it :D

You have a branch cut along the x-axes.

The result is \frac{\pi}{\sqrt{2}}

If you want I can scan you the procedure ^^

Plus how did you get to the conclusion that the function under the integral is odd? :\
 
Last edited:
Also the integral \int^{ +\infty }_{0}\frac{ t^2 \mbox{d} t }{ t^4+1 }=\frac{\pi }{2 \sqrt{2}}

While \int^{ +\infty }_{0}\frac{ t^2 \mbox{d} t }{ t^4+1 }=\frac{\pi }{\sqrt{2}}

That's why you need the branch cut...
 

Attachments

dingo_d said:
Also the integral \int^{ +\infty }_{0}\frac{ t^2 \mbox{d} t }{ t^4+1 }=\frac{\pi }{2 \sqrt{2}}

While \int^{ +\infty }_{0}\frac{ t^2 \mbox{d} t }{ t^4+1 }=\frac{\pi }{\sqrt{2}}

That's why you need the branch cut...

That was just a small mistake by the OP during the substitution where he left out a 2 (dx = 2 t \, dt). The correct integral (after substitution) should have been.

\int^{ +\infty }_{0}\frac{ 2 t^2 \mbox{d} t }{ t^4+1 }=\frac{\pi }{\sqrt{2}}

So actually no branch cut was required.

BTW. Normally in the homework section it's better to give hints and advice on how to proceed rather than answers or worked solutions.
 
Yeah, I had some time to spend...
 
Question: A clock's minute hand has length 4 and its hour hand has length 3. What is the distance between the tips at the moment when it is increasing most rapidly?(Putnam Exam Question) Answer: Making assumption that both the hands moves at constant angular velocities, the answer is ## \sqrt{7} .## But don't you think this assumption is somewhat doubtful and wrong?

Similar threads

  • · Replies 7 ·
Replies
7
Views
2K
  • · Replies 105 ·
4
Replies
105
Views
6K
Replies
2
Views
2K
  • · Replies 4 ·
Replies
4
Views
1K
  • · Replies 22 ·
Replies
22
Views
3K
  • · Replies 19 ·
Replies
19
Views
2K
  • · Replies 5 ·
Replies
5
Views
2K
  • · Replies 21 ·
Replies
21
Views
2K
  • · Replies 8 ·
Replies
8
Views
2K
  • · Replies 2 ·
Replies
2
Views
2K